Answer choice A
I understand we need to make an assumption between exercising and arthritis, but would that not m...
AndrewArabie on April 27 at 06:36PM
  • June 2013 LSAT
  • SEC1
  • Q5
2
Replies
many vs. some
hello, I understand the explanation provided for the correct answer choice when it says, "The two...
cmbaro on October 14, 2022
  • June 2013 LSAT
  • SEC1
  • Q21
2
Replies
Reasoning for C
I got this question right, but almost picked C. The reasoning given for why C is wrong is tha...
RachP on June 1, 2022
  • June 2013 LSAT
  • SEC1
  • Q11
1
Reply
How is A wrong?
I still don't know why B is correct. B to me looks like it's strengthening the argument.
Tyler808 on May 21, 2022
  • June 2013 LSAT
  • SEC1
  • Q22
1
Reply
I choose E as the correct answer, but what is w...
What is the difference between A and E? Is A wrong because its saying it will guarantee benefit t...
Tyler808 on May 19, 2022
  • June 2013 LSAT
  • SEC1
  • Q9
4
Replies
The conditional logic is fricken crazy
How is possible to diagram this within a certain time, negate the whole thing and find the answer...
Tyler808 on May 19, 2022
  • June 2013 LSAT
  • SEC1
  • Q12
2
Replies
What is the agreement here?
I read this over and over and in simpler terms, what is the agreement?
Tyler808 on May 17, 2022
  • June 2013 LSAT
  • SEC1
  • Q2
1
Reply
What's the flaw?
I read through the other threads for an explanation but came up lacking. I was able to get the...
ikarus on February 22, 2022
  • June 2013 LSAT
  • SEC1
  • Q14
5
Replies
Answer Choice C
The stimulus says that "amateur scientists have also provided many significant contributions." Ho...
MikeGreen on January 25, 2022
  • June 2013 LSAT
  • SEC1
  • Q21
3
Replies
Clarification
Could you please explain what makes (B) correct?
tselimovic on November 28, 2020
  • June 2013 LSAT
  • SEC1
  • Q22
3
Replies
Question
I'm having trouble understanding the reasoning behind the correct answer choice. If the worm did ...
cliu on October 14, 2020
  • June 2013 LSAT
  • SEC1
  • Q19
8
Replies
Developing skills to identify premises and conc...
Can you please explain why D is correct and C is incorrect
@Joshua on October 1, 2020
  • June 2013 LSAT
  • SEC1
  • Q4
1
Reply
June 2013 LSAT lr 17
Is C the correct answer because punching Bobo doesn't necessarily translate to punching other chi...
kens on August 3, 2020
  • June 2013 LSAT
  • SEC1
  • Q17
1
Reply
June 2013 LSAT 24
Can someone please explain why E is incorrect? Thanks in advance!
kens on August 3, 2020
  • June 2013 LSAT
  • SEC1
  • Q24
2
Replies
A vs. E
I was debating between A and E. Can someone explain why E is correct and A is wrong? Thanks!
colleen_ on August 1, 2020
  • June 2013 LSAT
  • SEC1
  • Q14
1
Reply
Confused between A and B
Hi there, Can someone please explain why B is a better choice than A? Thank you.
faithwood21 on July 14, 2020
  • June 2013 LSAT
  • SEC1
  • Q25
1
Reply
How can B be elimnated
I got the answer right but I struggled with B. It felt like something was missing but I'm not sur...
avif on June 24, 2020
  • June 2013 LSAT
  • SEC1
  • Q18
3
Replies
Answer E
Hi! Could you please explain why answer choice E is wrong? Thanks so much!
DavidW on June 19, 2020
  • June 2013 LSAT
  • SEC1
  • Q23
1
Reply
answer choice A
I understand the correct answer, and I find it to be the most correct. But-- why is answer choice...
annasc on June 18, 2020
  • June 2013 LSAT
  • SEC1
  • Q21
1
Reply
Confused
Can you Explain why the answer is E and not B?
samlopez1097 on June 14, 2020
  • June 2013 LSAT
  • SEC1
  • Q11
2
Replies